What is the volume of this figure?

Enter your answer in the box.

ft³

What Is The Volume Of This Figure?Enter Your Answer In The Box. Ft

Answers

Answer 1

Answer: 76 cubic feet

Step-by-step explanation:

Think about breaking it into two rectangular prisms:

Total volume = Volume of longer prism + volume of shorter prism

Total volume = 8x4x2 + 2x2x3

Total volume = 76 cubic feet

Answer 2
The Answer is 76ft.

Related Questions

Which relation is a function?

Answers

The only graph that represents a function is: Graph D

How to identify a function?

A function is defined as a relationship or expression that involves one or more variables. It typically has a set of input and outputs.  Each input has only one output. The function is the description of how the inputs relate to the output.

A function is a relation which describes that there should be only one output for each input (or) we can say that a special kind of relation (a set of ordered pairs), which follows a rule i.e., every X-value should be associated with only one y-value is called a function.

From the graphs, we can see that:

Graph A has 2 outputs at x = -2

Graph B has 2 outputs at x = 0

Graph C has two outputs at x = -1

Graph D has a unique output for every input and as such it is a function

Read more about function description at: https://brainly.com/question/24057830

#SPJ1

How do l do this? Help please

Answers

By expanding and simplifying the algebraic expression (y + 7)(y - 5), we have the result y² + 2y - 35

How to expand and simplify algebraic expression in brackets

To expand an algebraic expression in brackets, we need to use the distributive property to make is easy for simplification, thus we expand and simplify the expression as follows:

(y + 7)(y - 5) = y(y - 5) + 7(y - 5) {distributive property}

(y + 7)(y - 5) = y² - 5y + 7y - 35

by simplification, we have;

(y + 7)(y - 5) = y² + 2y - 35

Therefore, the expansion and simplification of the algebraic expression (y + 7)(y - 5), gives the result y² + 2y - 35.

Read more about algebra here:https://brainly.com/question/723406

#SPJ1

Please help !!!!!!!!!!!!!!!!

Answers

length LD = 6 units

length DF = 9 units

length HF = 6 units

length LH = 9 units

length L'D' = 2 units

length D'F' = 3 units

length H'F' = 2 units

length L'H' = 3 units

What is dilation?

Dilation is the scaling of an object, where it gets bigger or smaller.

Scale factor = new dimension/old dimension

length LD = 15-9 = 6units

length DF = 6-(-3) = 9units

length HF = 15-9 = 6 units

length LH = 6-(-3) = 9 units

Since the scale factor is 1/3, we divide the preimage dimension by 3 to get the dimensions of the new image.

learn more about dilation from

https://brainly.com/question/10253650

#SPJ1

There may be many polynomial equations with integer coefficient which have the square root of 8 - 1 and the square root of 8 + 1 as its roots. Construct an equation of smallest degree with integer coefficients that has the square root of 8 - 1 and the square root 8 + 1 as its roots

Answers

The equation of smallest degree with integer coefficients will be f(x) = x^2 - (3 + √7)x + 3√7

How to explain the equation

It should be noted that to begin, let us establish the following:

x = √(8 - 1) which is equal to √7.

y = √(8 + 1) which is equivalent to √9. Therefore, y equals 3.

For the value of x, its conjugate coincides with √7; hence:

(x - x') × (x - y') = (x - √7)(x - 3)

Equation expansion yields:

f(x) = x^2 - (3 + √7)x + 3√7

By substituting for both values x and y in the equation above, we can show that f(x) satisfies the given specifications:

When x takes on a value of √7:

f(√7) = (√7)^2 - (3 + √7)√7 + 3√7 = 7 - 7 = 0

With an input value of 3:

f(3) = 3^2 - (3 + √7)(3) + 3√7 = 9 - 9 - 3√7 + 3√7 = 0

The lowest degree polynomial with rational coefficients having √(8 - 1) and √(8 + 1) as its roots is: f(x) = x^2 - (3 + √7)x + 3√7

Learn more about equations on

https://brainly.com/question/2972832

#SPJ1

Rachel works at a lemonade stand at the park on Monday. She used 1 2/5 bags of lemons on Tuesday. She used 1 1/4 times as many lemons as on Monday. How many bags of lemons did Rachel use on Tuesday?

Answers

Let's start by finding how many bags of lemons Rachel used on Monday. We know she used a whole number of bags plus a fraction, so we'll need to convert the mixed number to an improper fraction:

1 2/5 = 7/5

So Rachel used 7/5 bags of lemons on Monday.

On Tuesday, Rachel used 1 1/4 times as many lemons as on Monday. To find out how many bags of lemons that is, we can multiply the amount Rachel used on Monday by 1 1/4:

1 1/4 = 5/4

5/4 times 7/5 = 35/20 = 7/4

So Rachel used 7/4 bags of lemons on Tuesday, which is the same as 1 3/4 bags of lemons.

Therefore, Rachel used 1 3/4 bags of lemons on Tuesday.

what is the value of sin B?
8/17
17/15
15/17
8/15​

Answers

The sine of angle B is obtained dividing the length of the opposite side to angle B by the length of the hypotenuse.

What are the trigonometric ratios?

The three trigonometric ratios are the sine, the cosine and the tangent, and they are defined on the bullet points below:

Sine of angle = length of opposite side to the angle divided by the length of the hypotenuse of the triangle.Cosine of angle = length of adjacent side to the angle divided by the length of the hypotenuse of the triangle.Tangent of angle = length of opposite side to the angle divided by the length of the adjacent side to the angle of the triangle.

Missing Information

The problem is incomplete, hence the general procedure to obtain the sine of angle B is presented.

More can be learned about trigonometric ratios at brainly.com/question/24349828

#SPJ1

In the month, what percent of Dinesh’s time was spent on Project X? Be sure to show your work, and answer the question below.

Answers

The percentage of Dinesh’s time spent on Project X is 36.90%.

We have,

From the table,

The total time spent on the X-project each week is given as:

= 23(1/3) + 16(1/3) + 9.33 + 4.20

= 23.33 + 16.33 + 9.33 + 4.20

= 53.19

This is the time spent on X-project for the month.

Now,

The total time spent on all the events in the table.

= 144.13

The percent of Dinesh’s time spent on Project X.

= 53.19/144.13 x 100

= 36.90%

Thus,

The percentage of Dinesh’s time spent on Project X is 36.90%.

Learn more about percentages here:

https://brainly.com/question/11403063

#SPJ1

what is this pls help

Answers

(3(3))to the power of 2 is 27
4(3) is 12
The answer should be 39
3(3)+4(3)

3(3)=9
9 squared is 9x9=81
4(3)=12

So your equation simplified is
81+12=93

Hope this helps

For a population growing at 3% a year, what is the yearly growth factor?

Answers

1.03 is the growth factor because you’re taking the original population and ADDING 3 percent.

LAST QUESTION FOR THE NIGHT I NEED HELP WITH IT I’M KINDA STRUGGLING HERE

Answers

The solution to the system of equations in this problem is given as follows:

(-1, -5).

How to solve the system of equations?

The system of equations in the context of this problem is defined as follows:

y = x - 4.y = -3x - 8.

The x-coordinate of the solution in the context of this problem is obtained equaling the two equations, as follows:

x - 4 = -3x - 8

4x = -4

x = -1.

Hence the y-coordinate of the solution is obtained as follows:

y = -1 - 4

y = -5.

Thus the solution is:

(-1, -5).

More can be learned about a system of equations at https://brainly.com/question/13729904

#SPJ1

The British government has a consol bond outstanding paying per year forever. Assume the current interest rate is per year.
a. What is the value of the bond immediately after a payment is​ made?
b. What is the value of the bond immediately before a payment is​ made?

Answers

a. The value of the bond immediately after a payment is made is the same as the payment divided by the interest rate. For example, if the bond pays £100 per year and the current interest rate is 5%, the value of the bond immediately after a payment is made is £100 / 0.05 = £2000.

b. The value of the bond immediately before a payment is made is the same as the value of the bond immediately after a payment is made minus the coupon payment. For example, if the bond pays £100 per year and the current interest rate is 5%, the value of the bond immediately before a payment is made is £2000 - £100 = £1900.

The admission fee at an amusement park is $2.25 for children and $5.60 for adults. On a certain day, 269 people entered the park, and the admission fees collected totaled 1091 dollars. How many children and how many adults were admitted?

Answers

A lot of them



sorry I’m very tired and can’t do this rn

Suppose that 3 items will be chosen from a group with 7 total items.
Given that order does not matter, how many different ways can the items be chosen?
A) 35
B)70
C)140
D)210

Answers

The number of ways of selecting 3 items from a total of 7 items, where order does not matter is (a) 35 ways.

The number of ways to choose three items from a group of seven, where order does not matter, is given by the combination formula:

⇒ C(n,r) = n!/(r! × (n-r)!),

where n is = total number of items, and r is = number of items to be chosen,

We have, total items (n) = 7 and items to be selected (r) = 3.

Substituting the values,

We get;

⇒ C(7,3) = 7!/(3! × (7-3)!),

⇒ 7!/(3! × 4!),

⇒ (7 × 6 × 5)/(3 × 2 × 1),

⇒ 35 ways.

Therefore, the correct option is (a).

Learn more about Selection here

https://brainly.com/question/28348278

#SPJ1

Select the correct answer. Determine which statement is true about the zeros of the function graphed below. An upward parabola f on a coordinate plane vertex at (1, 4) and intercepts the y-axis at 5 units. A. Function f has one real solution and one complex solution. B. Function f has exactly one real solution and no complex solutions. C. Function f has exactly two real solutions. D. Function f has exactly two complex solutions.

Answers

The correct option is D, the equation has two complex solutions.

Which is the correct statement about the quadratic equation?

Here we can see that we have the graph of a quadratic equation.

It opens upwards, and we can see that it has a vertex at (1, 4), which intercepts the y-axis at y = 5.

Now, we say that the solutions of a quadratic are the values of x such that the function becomes zero.

Particualrly, in this graph we can see that the graph never intercepts the x-axis, that means that this equation has no real roots.

Then the correct option is:

"Function f has exactly two complex solutions."

Learn more about quadratic equations at:

https://brainly.com/question/1214333

#SPJ1

help asap im so lost

Answers

Answer:250

Step-by-step explanation:

I think they just want you to multiply the 25 and the 10 (radius squared times the height)

After running a 100 meter dash, Jane turns left 40 degrees and walks 60 meter. True or false: she is closer now to her starting position than when she crossed the finish line. What about if Jane turns left 90 degrees or 120 degrees?

Answers

False. When Jane turns left 40 degrees and walks 60 meters, she is farther away from her starting position than when she crossed the finish line.

If Jane turns left 90 degrees or 120 degrees, then she will also be farther away from her starting position than when she crossed the finish line. Turning left 90 degrees or 120 degrees will take her in a different direction, and thus she will be farther away from her starting position than when she crossed the finish line.

Therefore, the given statement is false.

Learn more about the bearings here:

brainly.com/question/27962362.

#SPJ1

Select the correct answer. Which point lies on the circle represented by the equation (x − 3)2 + (y + 4)2 = 62? A. (9,-2) B. (0,11) C. (3,10) D. (-9,4) E. (-3,-4) Reset Next

Answers

Answer: The point that satisfies the equation is E. (-3, -4).

Explanation:

To determine which point lies on the circle represented by the equation (x − 3)² + (y + 4)² = 6², we can substitute the coordinates of each point into the equation and see which one satisfies the equation.

A. (9, -2): (9 − 3)² + (-2 + 4)² = 6²

(6)² + (2)² = 36

36 + 4 = 40 ≠ 36

B. (0, 11): (0 − 3)² + (11 + 4)² = 6²

(-3)² + (15)² = 36

9 + 225 = 234 ≠ 36

C. (3, 10): (3 − 3)² + (10 + 4)² = 6²

(0)² + (14)² = 36

0 + 196 = 196 ≠ 36

D. (-9, 4): (-9 − 3)² + (4 + 4)² = 6²

(-12)² + (8)² = 36

144 + 64 = 208 ≠ 36

E. (-3, -4): (-3 − 3)² + (-4 + 4)² = 6²

(-6)² + (0)² = 36

36 + 0 = 36

The point that satisfies the equation is E. (-3, -4).

PLEASE HURRY!!!!
The height h in feet of an object shot straight up with initial velocity v in feet per second is given by h = −16t^2 + vt + c, where c is the initial height of the object above the ground. A model rocket is shot vertically up from a height of 6 feet above the ground with an initial velocity of 22 feet per second. Will it reach a height of 10 feet? Identify the correct explanation for your answer.



A. No; The discriminant is positive so the rocket will reach a height of 10 feet.

B.Yes; The discriminant is positive, so the rocket will reach a height of 10 feet.

C.No; The discriminant is negative, so the rocket will not reach a height of 10 feet.

D.Yes; The discriminant is zero, so the rocket will reach a height of 10 feet.

Answers

Answer:

B.

Step-by-step explanation:

We can use the given formula to find the time it takes for the rocket to reach a height of 10 feet:

10 = -16t^2 + 22t + 6

Rewriting the equation in standard quadratic form:

16t^2 - 22t + 4 = 0

Using the quadratic formula:

t = (22 ± sqrt(22^2 - 4(16)(4)))/(2(16))

t = (22 ± sqrt(36))/32

t = 3/4 or 1/4

Since the rocket reaches a height of 10 feet at two different times (3/4 and 1/4 seconds), it must pass through that height twice during its flight. Therefore, the rocket will reach a height of 10 feet. The correct answer is B.

A rectangular prism has a volume of 2288 cubic meters a height of 13 meters and a length of 22 meters. What is the measure of the missing dimension?

Answers

The measure of the missing dimension is 8 meters.

We are given that;

Volume= 2288 cubic meters

Height= 13meter

Length= 22meter

Now,

To find the width.

You can rearrange the formula to solve for width by dividing both sides by length and height:

Width = Volume / (length × height)

Plug in the given values:

Width = 2288 / (22 × 13)

Simplify:

Width = 2288 / 286

Width = 8

Therefore, by the given rectangular prism the answer will be 8 meters.

Learn more about a rectangular prism;

https://brainly.com/question/21308574

#SPJ1

The endpoints of WX are W(-5,-1) and X(2,6).

What is the length of WX?
A. 7
B. 14
C. 4√2
D. 7/2

Answers

To find the length of WX, we can use the distance formula:

d = sqrt((x2 - x1)^2 + (y2 - y1)^2)

where (x1, y1) = (-5, -1) and (x2, y2) = (2, 6)

d = sqrt((2 - (-5))^2 + (6 - (-1))^2)
d = sqrt(49 + 49)
d = sqrt(98)
d = 7sqrt(2)

Therefore, the length of WX is C. 4√2.

The list represents the ages of students in a gymnastics class. 10, 10, 11, 12, 12, 13, 13, 14, 14, 15 If another student of age 15 joins the class, how is the mean affected? The mean will remain the same at 13. The mean will remain the same at about 12.4. The mean will increase to about 12.6. The mean will decrease to about 11.

Answers

The required mean will increase to about 12.6 when another student of age 15 joins the class, and the correct answer is C.

The original mean is the sum of the ages divided by the number of students:

Mean = (10 + 10 + 11 + 12 + 12 + 13 + 13 + 14 + 14 + 15) / 10

Mean = 124 / 10

Mean = 12.4

If another student of age 15 joins the class, the new sum of the ages is:

Sum = 10 + 10 + 11 + 12 + 12 + 13 + 13 + 14 + 14 + 15 + 15

Sum = 139

The new mean is the new sum of ages divided by the new number of students:

New mean = Sum / (Number of students + 1)

New mean = 139 / 11

New mean = 12.63636... or about 12.6 (rounded to one decimal place)

Therefore, the mean will increase to about 12.6 when another student of age 15 joins the class, and the correct answer is C.

Learn more about mean here:

https://brainly.com/question/15397049

#SPJ1

ASAP.
jack goes for a ride on a ferris wherl thst has a radius of 51 yards. the center of the ferris sherl is 61 yards above the ground. he starts bis rifr at the 9 oclock position and travels counter clockwise. define a function g that tepresents jacks verticL distance above the grihdn in yards in terms of the angel ( meassured in radians) jack has swept out measured grom the 9 oclock positions

Answers

Answer:

112 yards

Step-by-step explanation:

The center of the Ferris wheel is 61 yards above the ground and the radius is 51 yards. When Jack is at the 9 o'clock position, he is at a distance of 112 yards from the center of the Ferris wheel (51 yards from the center plus 61 yards above the ground). Let θ be the angle that Jack has swept out measured from the 9 o'clock position, in radians.

The function g that represents Jack's vertical distance above the ground in yards in terms of the angle θ is:

g(θ) = 61 + 51sin(θ)

where sin(θ) represents the vertical component of the distance Jack has traveled.

Note that when θ = 0, sin(θ) = 0, which means Jack is at the very top of the Ferris wheel, 112 yards above the ground. When θ = π/2, sin(θ) = 1, which means Jack is at the 12 o'clock position, 112 + 51 = 163 yards above the ground. Similarly, when θ = π, sin(θ) = 0, which means Jack is at the very bottom of the Ferris wheel, 112 yards above the ground.

Use a half-angle identity to find the exact value of tan5π/12

Answers

The required exact value of tan(5π/12) is (2 + √3) / 3.

We can use the half-angle identity for a tangent:

tan(x/2) = [1 - cos(x)] / sin(x)

to find the exact value of tan(5π/12), since 5π/12 is a half-angle of 5π/6.

First, we find the values of sin(5π/6) and cos(5π/6) using the unit circle:

sin(5π/6) = sin(π - π/6) = sin(π/6) = 1/2

cos(5π/6) = cos(π - π/6) = -cos(π/6) = -√3/2

Now we can use the half-angle identity for a tangent with x=5π/6:

tan(5π/12) = tan[(5π/6)/2] = [1 - cos(5π/6)] / sin(5π/6)

                = [1 - (-√3/2)] / (1/2)

               = (2 + √3) / 3

Therefore, the exact value of tan(5π/12) is (2 + √3) / 3.

Learn more about trig ratios here:

https://brainly.com/question/14977354

#SPJ1

A beaker is shaped like a cylinder with a radius of 1.8 inches and a height of 4.6 inches. It is filled to the top with a solution. Caleb wants to pour it into a different beaker with a radius of 1.25 inches. What is the minimum height the second beaker must be so it does not overflow? Round to the nearest tenth.

Answers

The minimum height of the second beaker must be approximately 13.5 inches to hold the solution without overflowing.

Now, For the minimum height of the second beaker, we need to find its volume first.

The volume of the first beaker is given by;

⇒ V₁ = πr₁²h₁

where r₁ is the radius and h₁ is the height.

Substituting the given values, we get:

V₁ = π(1.8)²(4.6)

V₁ ≈ 66.85 cubic inches

Since, the first beaker is filled to the top, its volume equals the volume of the solution.

Therefore, the volume of the solution is also 66.85 cubic inches.

To find the minimum height of the second beaker, we need to use the formula for the volume of a cylinder:

V₂ = πr₂²h₂

where r₂ is the radius and h₂ is the height of the second beaker.

We want to find h₂ such that V₂ is equal to 66.85 cubic inches.

Dividing both sides of the equation by πr₂², we get:

h₂ = V₂ / (πr₂²)

Substituting the given value for r₂, we get:

h₂ = 66.85 / (π(1.25)²)

h₂ ≈ 13.5 inches

Therefore, the minimum height of the second beaker must be approximately 13.5 inches to hold the solution without overflowing.

Learn more about the multiplication visit:

https://brainly.com/question/10873737

#SPJ1

Can someone help me with the 4 questions

Answers

7. The total number of shirts produced in an hour cannot exceed 1000. (E)
8. The total cost of producing dress shirts and casual shirts together cannot exceed $8400 per hour. (C)
9. The number of casual shirts produced per hour must be non-negative. (F)
10. The number of dress shirts produced per hour must be non-negative. (A)

Therefore, the correct matches are:
7. B
8. C
9. F
10. A

Solve the following for θ, in radians, where 0≤θ<2π.
−sin2(θ)−4sin(θ)+4=0
Select all that apply:

1.1
2.52
0.98
0.69
1.43
2.17

Answers

Answer:0.98

2.17 are correct

Step-by-step explanation:

-u^2 - 4u + 4 = 0

Multiplying both sides by -1, we get:

u^2 + 4u - 4 = 0

Now we can use the quadratic formula to solve for u:

u = (-b ± sqrt(b^2 - 4ac)) / 2a

where a = 1, b = 4, and c = -4. Substituting these values, we get:

u = (-4 ± sqrt(4^2 - 4(1)(-4))) / 2(1)

u = (-4 ± sqrt(32)) / 2

u = (-4 ± 4sqrt(2)) / 2

u = -2 ± 2sqrt(2)

Therefore, either:

Prove the following:

Answers

Based on the information, HH is a subset of Ha, which is a subset of aH, which is a subset of H, which is a subset of HH

How to make the proof

It should be noted that to demonstrate this theory, we must show that each of these collections are components of the others and consequently all are equivalent.

At first, let's evaluate HH. As H is a subgroup of G, it is dependent on multiplication so for any two elements h1, h2 in H, the product h1h2 is also in H. Thus, every entity within HH can be expressed as the aggregate of h1h2 for certain elements h1, h2 from H. Since H is a subgroup, it is also subject to inverse property, so h2^-1h1^-1 remains inside H. Clearly, h1h2 thus translates to h'(h2^-1h1^-1)h'', where h', h'' are members of H.

This illustrates that every component of HH can be presented as h'h'' for a pair of h', h'' which exist in H, therefore approximating with accuracy the definition of H. Thus, we have demonstrated that HH is part of H.

Leane more about subset on

https://brainly.com/question/13265691

#SPJ1

We can approximate the amount of current in amps I

Answers

The measure of the third outgoing current will be 0.5 amperes.

We know that,

The analysis of mathematical representations is algebra, and the handling of those symbols is logic.

The three incoming currents at a node in an electrical circuit measure 0.7 amps, 0.68 amps, and 0.47 amps two of the three outgoing currents measure 0.8 amps and 0.55 amps.

Then the measure of the third outgoing current will be

We know that the sum of the incoming current will be equal to the sum of the outgoing current at a junction.

Let the incoming current be I₁, I₂, and I₃. And the outgoing current is I₄, I₅, and I₆.

Then we have

I₁ + I₂ + I₃ = I₄ + I₅ + I₆

0.7 + 0.68 + 0.47 = 0.8 + 0.55 + I₆

1.85 = 1.35 + I₆

I₆ = 0.5

Thus, the measure of the third outgoing current will be 0.5 amperes.

More about the Algebra link is given below.

brainly.com/question/953809

#SPJ1

complete question:

The three incoming currents at a node in an electrical circuit measure 0.7 amps, 0.68 amps, and 0.47 amps two of the three outgoing current measure 0.8 amps and 0.55 amps find the measure of the third outgoing current

A toy ball can be modeled as a sphere. Moussa measures its circumference as 56.3 cm. Find the ball’s volume in cubic centimeters. Round your answer to the nearest tenth if necessary.

Answers

Answer:

  3013.5 cm³

Step-by-step explanation:

Given a ball with a circumference of 56.3 cm, you want to know its volume.

Radius

The formula for circumference is ...

  C = 2πr

Solving for radius, we get ...

  r = C/(2π) = 56.3 cm/(2π) ≈ 8.96042 cm

Volume

The volume of a sphere is given by ...

  V = 4/3πr³

  V = 4/3π(8.96042 cm)³ ≈ 3013.5 cm³

The ball's volume is about 3013.5 cubic centimeters.

__

Additional comment

Alternatively, we could use the formula ...

  V = C³/(6π²)

to get the same result.

<95141404393>

WHats this answer?
Please help!

Answers

The measure of angle 3 is 52⁰.

The measure of angle 5 is 38⁰.

The measure of angle 6 is 52⁰.

What is the measure of angle 3, 5 and 6?

The measure of angle 3, 5 and 6 is calculated as follows;

angle 1 + angle 2 + angle 3 = 180 (sum of angles in straight line )

90 + 38 + angle 3 = 180

angle 3 = 180 - 128

angle 3 = 52⁰

The measure of angle 5 is calculated as follows;

angle 5 = angle 2 (vertical opposite angles are equal)

angle 5 = 38⁰

The measure of angle 6 is calculated as follows;

angle 6 = angle 3 (vertical opposite angles are equal)

angle 6 = 52⁰

Learn more about angles on a straight line here: https://brainly.com/question/24024505

#SPJ1

Other Questions
describe the advantages of the 3x3 cross tie configuration? Which blood vessel bifurcates into the right and left common iliac arteries? Most economists agree that the immediate cause of the majority of cyclical change level of real output is unexpected changes in the:A) level of total spending, and aggregate demand.B) rate of unemployment.C) rate of inflation.D) stock market price indexes. How many different subsets of $\{1, 2, 3, 4, 5, 6, 7, 8, 9, 10, 11, 12\}$ contain at least one element in common with each of the sets $\{2, 4, 6, 8, 10, 12\}$, $\{3, 6, 9, 12\}$ and $\{2, 3, 5, 7, 11\}\,?$ bjs is a company that hand-makes jewelry boxes. forecasts of sales for next year are 150 in autumn, 400 in winter, and 50 in spring. they purchase plain jewelry boxes for $20 each and incur a holding cost of $1.2 per quarter per item. bjs hires part-time art students to craft their designs, paying them $5.50 per hour in autumn, $7.00 per hour in winter, and $6.25 per hour in spring. each jewelry box takes two hours to complete, and the company can rent equipment for $65 for three months, which can be returned if no production occurs in a quarter. the goal is to determine the optimal number of jewelry boxes to produce in each quarter to minimize production cost, inventory holding cost, and rental cost. Name the syndrome Seen in premature babies whose lungs have not matured enough to produce surfactant? A. What are the differences of time basis and piece rate basis system?B. Nora is a Business Analytics Manager at Namshi a multinational E-commerce company. She is paid OMR 25 per hour based on hourly rate, for 40 hours per week. According, to the agreement, she is eligible for overtime (time-and-a-half pay). Calculate the following.4. If Nora worked for 58 hours in a week, what is her gross pay?5. What is her overtime rate per hour?6. What is her total overtime allowance? The water deprivation test is used to diagnose diabetes insipidus.TrueFalse louis owns several abandoned buildings in the city where drug dealers and homeless people are now living unlawfully. louis offers al $5,000 to burn down the buildings in order to run off the people living there. al sets fire to the buildings about a week later, but the fire department comes before much damage is done and now the drug dealers and homeless people are back. louis refuses to pay al because he claims al did not complete the job and the buildings are still standing. which of the following statements is true? what should the nurse aide do to communicate with a client who speaks and understands a foreign language that the nurse does not know which of the following actions should tiana take if she needs each line after the first line in a paragraph to be indented? What is the probability that either event will occur?Now, find the probability of event B.A6206B20P(B) = [?]Enter as a decimal rounded to the nearest hundredth.Enter 7. What distinctions does Macbeth make about men? How does Macbeth try to use this information to persuade themurderer? Which of the following terms refers to broadening of a light pulse as it propagates through a fiber strand?a. Pulse shapingb. Diffusionc. Absorptiond. Dispersion Prepare Cash Budget From Budgeted Transactions McCall Company anticipates a cash balance L04 of S100.000 on July 1, 2016.The following budgeted transactions for July 2016 present data related to anticipated cash receipts and cash disbursements: 1. For July, budgeted cash sales are $72,000 and budgeted credit sales are $600.000. (Credit sales for June were $550,000.) In the month of sale, 40% of credit sales are collected, with the balance collected in the month following sale. is paid for in the following month. 3. Budgeted cash disbursements for salaries and operating expenses for July total $200,000 5. A $25.000 income tax deposit is due to the federal government during July. 6. Depreciation for July will be S600. Prepare a cash budget for McCall Company for the month of July 2016. draw the organic products formed in the following reaction. do not specify the stereochemistry of the product. Apply the Ratio Test to determine convergence or divergence, or state that the Ratio Test is inconclusive. N = 1 2n/n! Jason worked 3 hours more than Keith. Jason worked 12 hours. Which equation represents this situation, if k is the numbers of hours Keith worked? Assume the sample is a random sample from a distribution that is reasonably normally distributed and we are doing inference for a sample mean. Find endpoints of at-distribution with 1% beyond them in each tail if the sample has size n=18.Round your answer to three decimal places. ) In terms of ad spending by media in the United States, the largest category is:A) televisionB) magazinesC) the InternetD) newspapers